Difference between revisions of "2018 AMC 12B Problems/Problem 15"

(Problem)
m (Solution 1 (For Dummies))
(9 intermediate revisions by 8 users not shown)
Line 4: Line 4:
 
<math>\textbf{(A) } 96 \qquad \textbf{(B) } 97 \qquad \textbf{(C) } 98 \qquad \textbf{(D) } 102 \qquad \textbf{(E) } 120 </math>
 
<math>\textbf{(A) } 96 \qquad \textbf{(B) } 97 \qquad \textbf{(C) } 98 \qquad \textbf{(D) } 102 \qquad \textbf{(E) } 120 </math>
  
== Solution 1 (For Dummies) ==
+
== Solution 1==
 
Analyze that the three-digit integers divisible by <math>3</math> start from <math>102</math>. In the <math>200</math>'s, it starts from <math>201</math>. In the <math>300</math>'s, it starts from <math>300</math>. We see that the units digits is <math>0, 1, </math> and <math>2.</math>
 
Analyze that the three-digit integers divisible by <math>3</math> start from <math>102</math>. In the <math>200</math>'s, it starts from <math>201</math>. In the <math>300</math>'s, it starts from <math>300</math>. We see that the units digits is <math>0, 1, </math> and <math>2.</math>
  
Write out the 1- and 2-digit multiples of <math>3</math> starting from <math>0, 1,</math> and <math>2.</math> Count up the ones that meet the conditions. Then, add up and multiply by <math>3</math>, since there are three sets of three from <math>1</math> to <math>9.</math> Then, subtract the amount that started from <math>0</math>, since the <math>300</math>'s all contain the digit <math>3</math>.
+
Write out the 1- and 2-digit multiples of <math>3</math> starting from <math>0, 1,</math> and <math>2.</math> Count up the ones that meet the conditions. Then, add up and multiply by <math>3</math>, since there are three sets of three from <math>1</math> to <math>9.</math> Then, subtract the amount that started from <math>0</math>, since the <math>300</math>'s ll contain the digit <math>3</math>.
  
 
We get: <cmath>3(12+12+12)-12.</cmath>
 
We get: <cmath>3(12+12+12)-12.</cmath>
Line 16: Line 16:
  
 
There are <math>4</math> choices for the last digit (<math>1, 5, 7, 9</math>), and <math>8</math> choices for the first digit (exclude <math>0</math>). We know what the second digit mod  <math>3</math> is, so there are <math>3</math> choices for it (pick from one of the sets <math>\{0, 6, 9\},\{1, 4, 7\}, \{2, 5, 8\}</math>). The answer is <math>4\cdot 8 \cdot 3 = \boxed{96}</math> (Plasma_Vortex)
 
There are <math>4</math> choices for the last digit (<math>1, 5, 7, 9</math>), and <math>8</math> choices for the first digit (exclude <math>0</math>). We know what the second digit mod  <math>3</math> is, so there are <math>3</math> choices for it (pick from one of the sets <math>\{0, 6, 9\},\{1, 4, 7\}, \{2, 5, 8\}</math>). The answer is <math>4\cdot 8 \cdot 3 = \boxed{96}</math> (Plasma_Vortex)
 +
 +
== Solution 3==
 +
 +
Consider the number of <math>2</math>-digit numbers that do not contain the digit <math>3</math>, which is <math>90-18=72</math>. For any of these <math>2</math>-digit numbers, we can append <math>1,5,7,</math> or <math>9</math> to reach a desirable <math>3</math>-digit number. However, <math>1 \equiv 7 \equiv 1</math> <math>(mod</math> <math>3)</math>, and thus we need to count any <math>2</math>-digit number <math>\equiv 2</math> <math>(mod</math> <math>3)</math> twice. There are <math>(98-11)/3+1=30</math> total such numbers that have remainder <math>2</math>, but <math>6</math> of them <math>(23,32,35,38,53,83)</math> contain <math>3</math>, so the number we want is <math>30-6=24</math>. Therefore, the final answer is <math>72+24= \boxed{96}</math>.
 +
 +
==Solution 4==
 +
Note that this isn't a great solution, but a more practical one to achieve the answer.
 +
 +
Notice that there are <math>300</math> numbers that have <math>3</math> digits and are divisible by <math>3</math> (from <math>102</math> to <math>999</math>). Now one by one apply the restrictions.
 +
 +
The restriction for only odd numbers would mean that half the numbers are taken out <math>\Rightarrow 300*\frac{1}{2} = 150</math>.
 +
 +
Next, apply the restriction of no <math>3</math>s. For the units digit, that would mean multiplying by <math>\frac{4}{5}</math> (remember that now you only have odd numbers to choose from).
 +
 +
For the tens that would mean multiplying by <math>\frac{9}{10}</math>, and for the hundreds that would mean multiplying by <math>\frac{8}{9}</math> (because you cant have 0 here).
 +
 +
Thus, we get <math>150*\frac{4}{5}*\frac{9}{10}*\frac{8}{9}=96</math>, which is <math>\boxed{A}</math>.
 +
 +
Sol by IronicNinja~
  
 
==See Also==
 
==See Also==
Line 21: Line 40:
 
{{AMC12 box|year=2018|ab=B|num-b=14|num-a=16}}
 
{{AMC12 box|year=2018|ab=B|num-b=14|num-a=16}}
 
{{MAA Notice}}
 
{{MAA Notice}}
 +
 +
[[Category:Introductory Combinatorics Problems]]
 +
[[Category:Introductory Number Theory Problems]]

Revision as of 18:18, 9 March 2020

Problem

How many odd positive 3-digit integers are divisible by 3 but do not contain the digit 3?

$\textbf{(A) } 96 \qquad \textbf{(B) } 97 \qquad \textbf{(C) } 98 \qquad \textbf{(D) } 102 \qquad \textbf{(E) } 120$

Solution 1

Analyze that the three-digit integers divisible by $3$ start from $102$. In the $200$'s, it starts from $201$. In the $300$'s, it starts from $300$. We see that the units digits is $0, 1,$ and $2.$

Write out the 1- and 2-digit multiples of $3$ starting from $0, 1,$ and $2.$ Count up the ones that meet the conditions. Then, add up and multiply by $3$, since there are three sets of three from $1$ to $9.$ Then, subtract the amount that started from $0$, since the $300$'s ll contain the digit $3$.

We get: \[3(12+12+12)-12.\]

This gives us: \[\boxed{\textbf{(A) } 96}.\]

Solution 2

There are $4$ choices for the last digit ($1, 5, 7, 9$), and $8$ choices for the first digit (exclude $0$). We know what the second digit mod $3$ is, so there are $3$ choices for it (pick from one of the sets $\{0, 6, 9\},\{1, 4, 7\}, \{2, 5, 8\}$). The answer is $4\cdot 8 \cdot 3 = \boxed{96}$ (Plasma_Vortex)

Solution 3

Consider the number of $2$-digit numbers that do not contain the digit $3$, which is $90-18=72$. For any of these $2$-digit numbers, we can append $1,5,7,$ or $9$ to reach a desirable $3$-digit number. However, $1 \equiv 7 \equiv 1$ $(mod$ $3)$, and thus we need to count any $2$-digit number $\equiv 2$ $(mod$ $3)$ twice. There are $(98-11)/3+1=30$ total such numbers that have remainder $2$, but $6$ of them $(23,32,35,38,53,83)$ contain $3$, so the number we want is $30-6=24$. Therefore, the final answer is $72+24= \boxed{96}$.

Solution 4

Note that this isn't a great solution, but a more practical one to achieve the answer.

Notice that there are $300$ numbers that have $3$ digits and are divisible by $3$ (from $102$ to $999$). Now one by one apply the restrictions.

The restriction for only odd numbers would mean that half the numbers are taken out $\Rightarrow 300*\frac{1}{2} = 150$.

Next, apply the restriction of no $3$s. For the units digit, that would mean multiplying by $\frac{4}{5}$ (remember that now you only have odd numbers to choose from).

For the tens that would mean multiplying by $\frac{9}{10}$, and for the hundreds that would mean multiplying by $\frac{8}{9}$ (because you cant have 0 here).

Thus, we get $150*\frac{4}{5}*\frac{9}{10}*\frac{8}{9}=96$, which is $\boxed{A}$.

Sol by IronicNinja~

See Also

2018 AMC 12B (ProblemsAnswer KeyResources)
Preceded by
Problem 14
Followed by
Problem 16
1 2 3 4 5 6 7 8 9 10 11 12 13 14 15 16 17 18 19 20 21 22 23 24 25
All AMC 12 Problems and Solutions

The problems on this page are copyrighted by the Mathematical Association of America's American Mathematics Competitions. AMC logo.png